Difference between revisions of "1994 AJHSME Problems/Problem 1"

 
(2 intermediate revisions by 2 users not shown)
Line 4: Line 4:
 
<math>\text{(A)}\ \dfrac{1}{3} \qquad \text{(B)}\ \dfrac{1}{4} \qquad \text{(C)}\ \dfrac{3}{8} \qquad \text{(D)}\ \dfrac{5}{12} \qquad \text{(E)}\ \dfrac{7}{24}</math>
 
<math>\text{(A)}\ \dfrac{1}{3} \qquad \text{(B)}\ \dfrac{1}{4} \qquad \text{(C)}\ \dfrac{3}{8} \qquad \text{(D)}\ \dfrac{5}{12} \qquad \text{(E)}\ \dfrac{7}{24}</math>
  
==Solution==
+
==Solution 1==
C and D are the only answer choices where the numerator is close to half of the denominator. <math>\dfrac{3}{8}</math> is equal to .375 and <math>\dfrac{5}{12}</math> is equal to approximately .41. <math>\boxed{\text{D}}</math>
+
C and D are the only answer choices where the numerator is close to half of the denominator. <math>\dfrac{3}{8} = .375</math> and <math>\dfrac{5}{12} \approx .41</math>. Thus the answer is <math>\boxed{\text{(D)}\ \frac{5}{12}}</math>
 +
 
 +
==Solution 2==
 +
Multiply every answer choice by <math>24</math>, the [[LCM]] of all of the answer choices. This gives
 +
 
 +
<math>\text{(A)}\ 8 \qquad \text{(B)}\ 6 \qquad \text{(C)}\ 9 \qquad \text{(D)}\ 10 \qquad \text{(E)}\ 7</math>
 +
 
 +
of which <math>\boxed{D}</math> is the largest.
 +
 
 +
==See Also==
 +
{{AJHSME box|year=1994|before=First <br /> Problem|num-a=2}}
 +
{{MAA Notice}}

Latest revision as of 11:47, 27 June 2023

Problem

Which of the following is the largest?

$\text{(A)}\ \dfrac{1}{3} \qquad \text{(B)}\ \dfrac{1}{4} \qquad \text{(C)}\ \dfrac{3}{8} \qquad \text{(D)}\ \dfrac{5}{12} \qquad \text{(E)}\ \dfrac{7}{24}$

Solution 1

C and D are the only answer choices where the numerator is close to half of the denominator. $\dfrac{3}{8} = .375$ and $\dfrac{5}{12} \approx .41$. Thus the answer is $\boxed{\text{(D)}\ \frac{5}{12}}$

Solution 2

Multiply every answer choice by $24$, the LCM of all of the answer choices. This gives

$\text{(A)}\ 8 \qquad \text{(B)}\ 6 \qquad \text{(C)}\ 9 \qquad \text{(D)}\ 10 \qquad \text{(E)}\ 7$

of which $\boxed{D}$ is the largest.

See Also

1994 AJHSME (ProblemsAnswer KeyResources)
Preceded by
First
Problem
Followed by
Problem 2
1 2 3 4 5 6 7 8 9 10 11 12 13 14 15 16 17 18 19 20 21 22 23 24 25
All AJHSME/AMC 8 Problems and Solutions

The problems on this page are copyrighted by the Mathematical Association of America's American Mathematics Competitions. AMC logo.png